Converse of statements about the rank of adjugate matrices












0












$begingroup$


Let the adjugate of a matrix be defined as the transpose of the cofactor matrix, denoted $A^{*}$. (Also termed the Classical Adjoint)



It can be proven that for any $ntimes n$ matrix $A$,



1) if rank($A$)= $n$ then rank(adj($A$)) = $n$



2) if rank($A$) = $n-1$ then rank(adj($A$)) = $1$



2) if rank($A$) < $n-1$ then rank(adj($A$)) = $0$



Suppose we were given an arbitrary $ntimes n$ matrix $A$. If we know that rank((adj($A$)) = $1$, does it necessarily imply that the rank of $A$ is $n-1$? For example, suppose $A$ has an unknown entry but it is given that rank(adj($A$)) =$1$, is it a valid approach to conclude that the rank of $A$ is $n-1$ and proceed to determine the unknown value by solving $det(A)=0$?



Similarly for the other two statements. Are they single direction statements or are they really if and only if statements?










share|cite|improve this question











$endgroup$












  • $begingroup$
    I'm not familiar with the statement you posted. What is your definition of adjoint matrix? Either $operatorname{rank} A = n$, $operatorname{rank} A = n-1$, or $operatorname{rank} A < n-1$. Assuming your statement is correct, if $operatorname{rank}operatorname{adj} A = 1$ and $operatorname{rank} A neq n-1$ you have a contradiction so $operatorname{rank}(A)$ must be $n-1$.
    $endgroup$
    – tch
    Dec 30 '18 at 15:23












  • $begingroup$
    @TylerChen : thank you for the reply, an adjoint matrix is the conjugate transpose of a matrix. I updated the post with some queries.
    $endgroup$
    – NetUser5y62
    Dec 30 '18 at 15:35










  • $begingroup$
    How do you use that definition of $A$ is not invertible? In any case, I don't think that is the definition of conjugate transpose.
    $endgroup$
    – tch
    Dec 30 '18 at 15:53










  • $begingroup$
    @TylerChen thank you for correcting that, I didn’t realize that was just a property thats applies when $A$ is invertible. I have removed it from the definition.
    $endgroup$
    – NetUser5y62
    Dec 30 '18 at 16:02
















0












$begingroup$


Let the adjugate of a matrix be defined as the transpose of the cofactor matrix, denoted $A^{*}$. (Also termed the Classical Adjoint)



It can be proven that for any $ntimes n$ matrix $A$,



1) if rank($A$)= $n$ then rank(adj($A$)) = $n$



2) if rank($A$) = $n-1$ then rank(adj($A$)) = $1$



2) if rank($A$) < $n-1$ then rank(adj($A$)) = $0$



Suppose we were given an arbitrary $ntimes n$ matrix $A$. If we know that rank((adj($A$)) = $1$, does it necessarily imply that the rank of $A$ is $n-1$? For example, suppose $A$ has an unknown entry but it is given that rank(adj($A$)) =$1$, is it a valid approach to conclude that the rank of $A$ is $n-1$ and proceed to determine the unknown value by solving $det(A)=0$?



Similarly for the other two statements. Are they single direction statements or are they really if and only if statements?










share|cite|improve this question











$endgroup$












  • $begingroup$
    I'm not familiar with the statement you posted. What is your definition of adjoint matrix? Either $operatorname{rank} A = n$, $operatorname{rank} A = n-1$, or $operatorname{rank} A < n-1$. Assuming your statement is correct, if $operatorname{rank}operatorname{adj} A = 1$ and $operatorname{rank} A neq n-1$ you have a contradiction so $operatorname{rank}(A)$ must be $n-1$.
    $endgroup$
    – tch
    Dec 30 '18 at 15:23












  • $begingroup$
    @TylerChen : thank you for the reply, an adjoint matrix is the conjugate transpose of a matrix. I updated the post with some queries.
    $endgroup$
    – NetUser5y62
    Dec 30 '18 at 15:35










  • $begingroup$
    How do you use that definition of $A$ is not invertible? In any case, I don't think that is the definition of conjugate transpose.
    $endgroup$
    – tch
    Dec 30 '18 at 15:53










  • $begingroup$
    @TylerChen thank you for correcting that, I didn’t realize that was just a property thats applies when $A$ is invertible. I have removed it from the definition.
    $endgroup$
    – NetUser5y62
    Dec 30 '18 at 16:02














0












0








0





$begingroup$


Let the adjugate of a matrix be defined as the transpose of the cofactor matrix, denoted $A^{*}$. (Also termed the Classical Adjoint)



It can be proven that for any $ntimes n$ matrix $A$,



1) if rank($A$)= $n$ then rank(adj($A$)) = $n$



2) if rank($A$) = $n-1$ then rank(adj($A$)) = $1$



2) if rank($A$) < $n-1$ then rank(adj($A$)) = $0$



Suppose we were given an arbitrary $ntimes n$ matrix $A$. If we know that rank((adj($A$)) = $1$, does it necessarily imply that the rank of $A$ is $n-1$? For example, suppose $A$ has an unknown entry but it is given that rank(adj($A$)) =$1$, is it a valid approach to conclude that the rank of $A$ is $n-1$ and proceed to determine the unknown value by solving $det(A)=0$?



Similarly for the other two statements. Are they single direction statements or are they really if and only if statements?










share|cite|improve this question











$endgroup$




Let the adjugate of a matrix be defined as the transpose of the cofactor matrix, denoted $A^{*}$. (Also termed the Classical Adjoint)



It can be proven that for any $ntimes n$ matrix $A$,



1) if rank($A$)= $n$ then rank(adj($A$)) = $n$



2) if rank($A$) = $n-1$ then rank(adj($A$)) = $1$



2) if rank($A$) < $n-1$ then rank(adj($A$)) = $0$



Suppose we were given an arbitrary $ntimes n$ matrix $A$. If we know that rank((adj($A$)) = $1$, does it necessarily imply that the rank of $A$ is $n-1$? For example, suppose $A$ has an unknown entry but it is given that rank(adj($A$)) =$1$, is it a valid approach to conclude that the rank of $A$ is $n-1$ and proceed to determine the unknown value by solving $det(A)=0$?



Similarly for the other two statements. Are they single direction statements or are they really if and only if statements?







linear-algebra matrices






share|cite|improve this question















share|cite|improve this question













share|cite|improve this question




share|cite|improve this question








edited Dec 31 '18 at 4:08







NetUser5y62

















asked Dec 30 '18 at 15:08









NetUser5y62NetUser5y62

525215




525215












  • $begingroup$
    I'm not familiar with the statement you posted. What is your definition of adjoint matrix? Either $operatorname{rank} A = n$, $operatorname{rank} A = n-1$, or $operatorname{rank} A < n-1$. Assuming your statement is correct, if $operatorname{rank}operatorname{adj} A = 1$ and $operatorname{rank} A neq n-1$ you have a contradiction so $operatorname{rank}(A)$ must be $n-1$.
    $endgroup$
    – tch
    Dec 30 '18 at 15:23












  • $begingroup$
    @TylerChen : thank you for the reply, an adjoint matrix is the conjugate transpose of a matrix. I updated the post with some queries.
    $endgroup$
    – NetUser5y62
    Dec 30 '18 at 15:35










  • $begingroup$
    How do you use that definition of $A$ is not invertible? In any case, I don't think that is the definition of conjugate transpose.
    $endgroup$
    – tch
    Dec 30 '18 at 15:53










  • $begingroup$
    @TylerChen thank you for correcting that, I didn’t realize that was just a property thats applies when $A$ is invertible. I have removed it from the definition.
    $endgroup$
    – NetUser5y62
    Dec 30 '18 at 16:02


















  • $begingroup$
    I'm not familiar with the statement you posted. What is your definition of adjoint matrix? Either $operatorname{rank} A = n$, $operatorname{rank} A = n-1$, or $operatorname{rank} A < n-1$. Assuming your statement is correct, if $operatorname{rank}operatorname{adj} A = 1$ and $operatorname{rank} A neq n-1$ you have a contradiction so $operatorname{rank}(A)$ must be $n-1$.
    $endgroup$
    – tch
    Dec 30 '18 at 15:23












  • $begingroup$
    @TylerChen : thank you for the reply, an adjoint matrix is the conjugate transpose of a matrix. I updated the post with some queries.
    $endgroup$
    – NetUser5y62
    Dec 30 '18 at 15:35










  • $begingroup$
    How do you use that definition of $A$ is not invertible? In any case, I don't think that is the definition of conjugate transpose.
    $endgroup$
    – tch
    Dec 30 '18 at 15:53










  • $begingroup$
    @TylerChen thank you for correcting that, I didn’t realize that was just a property thats applies when $A$ is invertible. I have removed it from the definition.
    $endgroup$
    – NetUser5y62
    Dec 30 '18 at 16:02
















$begingroup$
I'm not familiar with the statement you posted. What is your definition of adjoint matrix? Either $operatorname{rank} A = n$, $operatorname{rank} A = n-1$, or $operatorname{rank} A < n-1$. Assuming your statement is correct, if $operatorname{rank}operatorname{adj} A = 1$ and $operatorname{rank} A neq n-1$ you have a contradiction so $operatorname{rank}(A)$ must be $n-1$.
$endgroup$
– tch
Dec 30 '18 at 15:23






$begingroup$
I'm not familiar with the statement you posted. What is your definition of adjoint matrix? Either $operatorname{rank} A = n$, $operatorname{rank} A = n-1$, or $operatorname{rank} A < n-1$. Assuming your statement is correct, if $operatorname{rank}operatorname{adj} A = 1$ and $operatorname{rank} A neq n-1$ you have a contradiction so $operatorname{rank}(A)$ must be $n-1$.
$endgroup$
– tch
Dec 30 '18 at 15:23














$begingroup$
@TylerChen : thank you for the reply, an adjoint matrix is the conjugate transpose of a matrix. I updated the post with some queries.
$endgroup$
– NetUser5y62
Dec 30 '18 at 15:35




$begingroup$
@TylerChen : thank you for the reply, an adjoint matrix is the conjugate transpose of a matrix. I updated the post with some queries.
$endgroup$
– NetUser5y62
Dec 30 '18 at 15:35












$begingroup$
How do you use that definition of $A$ is not invertible? In any case, I don't think that is the definition of conjugate transpose.
$endgroup$
– tch
Dec 30 '18 at 15:53




$begingroup$
How do you use that definition of $A$ is not invertible? In any case, I don't think that is the definition of conjugate transpose.
$endgroup$
– tch
Dec 30 '18 at 15:53












$begingroup$
@TylerChen thank you for correcting that, I didn’t realize that was just a property thats applies when $A$ is invertible. I have removed it from the definition.
$endgroup$
– NetUser5y62
Dec 30 '18 at 16:02




$begingroup$
@TylerChen thank you for correcting that, I didn’t realize that was just a property thats applies when $A$ is invertible. I have removed it from the definition.
$endgroup$
– NetUser5y62
Dec 30 '18 at 16:02










2 Answers
2






active

oldest

votes


















1












$begingroup$

1)Yes. If $mathrm{rank}(mathrm{adj}(A))=n$ then $mathrm{adj}(A)$ is invertible, so $A=mathrm{det}(A)mathrm{adj}(A)^{-1}$ is invertible, because it can not be zero.



2)Yes. If $mathrm{rank}(mathrm{adj}(A))=1$ then $A$ is not invertible so $mathrm{rank}(A)leq n-1$ but by 3) we can not have $mathrm{rank}(A)< n-1$, since it would imply $mathrm{adj}(A)=0$. Thus $mathrm{rank}(A)= n-1$.



3)Yes. If $mathrm{adj}(A)=0$ then all $(n−1)×(n−1)$ minors of $A$ are zero, hence $mathrm{rank}(A)leq 2$






share|cite|improve this answer











$endgroup$





















    0












    $begingroup$

    If by adjoint of $A$ you mean adjoint operator, i.e. $operatorname{adj}(A) = A^*$ then your claimed properties are not true. For instance, any Hermetian matrix satisfies $A=A^*$ so the rank of $A$ is the rank of $A^*$.



    More generally, for any matrix $A$, the conjugate transpose $A^*$ always has the same rank.



    As an explicit example, consider the $3times 3$ matrix:
    $$
    A = begin{bmatrix}
    1 & 0 & 0 \
    0 & 1 & 0 \
    0 & 0 & 0
    end{bmatrix}
    $$



    This is clearly rank $2$, but the conjugate transpose is also rank 2.



    That said, if there exists some definition of "adjoint" satisfying the 3 properties you stated, then it is true that $operatorname{rank}operatorname{adj}(A)=1$ implies $operatorname{rank}(A) = n-1$. To see this, note that if $operatorname{rank}(A)$ did not equal $n-1$ then $operatorname{rank}operatorname{adj}(A)neq1$






    share|cite|improve this answer











    $endgroup$













    • $begingroup$
      $mathrm{adj}(A)$ is the transpose of the cofactor matrix of $A$ which make it different from $A^*$ the transpose of the conjugate.
      $endgroup$
      – mouthetics
      Dec 31 '18 at 15:15












    Your Answer








    StackExchange.ready(function() {
    var channelOptions = {
    tags: "".split(" "),
    id: "69"
    };
    initTagRenderer("".split(" "), "".split(" "), channelOptions);

    StackExchange.using("externalEditor", function() {
    // Have to fire editor after snippets, if snippets enabled
    if (StackExchange.settings.snippets.snippetsEnabled) {
    StackExchange.using("snippets", function() {
    createEditor();
    });
    }
    else {
    createEditor();
    }
    });

    function createEditor() {
    StackExchange.prepareEditor({
    heartbeatType: 'answer',
    autoActivateHeartbeat: false,
    convertImagesToLinks: true,
    noModals: true,
    showLowRepImageUploadWarning: true,
    reputationToPostImages: 10,
    bindNavPrevention: true,
    postfix: "",
    imageUploader: {
    brandingHtml: "Powered by u003ca class="icon-imgur-white" href="https://imgur.com/"u003eu003c/au003e",
    contentPolicyHtml: "User contributions licensed under u003ca href="https://creativecommons.org/licenses/by-sa/3.0/"u003ecc by-sa 3.0 with attribution requiredu003c/au003e u003ca href="https://stackoverflow.com/legal/content-policy"u003e(content policy)u003c/au003e",
    allowUrls: true
    },
    noCode: true, onDemand: true,
    discardSelector: ".discard-answer"
    ,immediatelyShowMarkdownHelp:true
    });


    }
    });














    draft saved

    draft discarded


















    StackExchange.ready(
    function () {
    StackExchange.openid.initPostLogin('.new-post-login', 'https%3a%2f%2fmath.stackexchange.com%2fquestions%2f3056904%2fconverse-of-statements-about-the-rank-of-adjugate-matrices%23new-answer', 'question_page');
    }
    );

    Post as a guest















    Required, but never shown

























    2 Answers
    2






    active

    oldest

    votes








    2 Answers
    2






    active

    oldest

    votes









    active

    oldest

    votes






    active

    oldest

    votes









    1












    $begingroup$

    1)Yes. If $mathrm{rank}(mathrm{adj}(A))=n$ then $mathrm{adj}(A)$ is invertible, so $A=mathrm{det}(A)mathrm{adj}(A)^{-1}$ is invertible, because it can not be zero.



    2)Yes. If $mathrm{rank}(mathrm{adj}(A))=1$ then $A$ is not invertible so $mathrm{rank}(A)leq n-1$ but by 3) we can not have $mathrm{rank}(A)< n-1$, since it would imply $mathrm{adj}(A)=0$. Thus $mathrm{rank}(A)= n-1$.



    3)Yes. If $mathrm{adj}(A)=0$ then all $(n−1)×(n−1)$ minors of $A$ are zero, hence $mathrm{rank}(A)leq 2$






    share|cite|improve this answer











    $endgroup$


















      1












      $begingroup$

      1)Yes. If $mathrm{rank}(mathrm{adj}(A))=n$ then $mathrm{adj}(A)$ is invertible, so $A=mathrm{det}(A)mathrm{adj}(A)^{-1}$ is invertible, because it can not be zero.



      2)Yes. If $mathrm{rank}(mathrm{adj}(A))=1$ then $A$ is not invertible so $mathrm{rank}(A)leq n-1$ but by 3) we can not have $mathrm{rank}(A)< n-1$, since it would imply $mathrm{adj}(A)=0$. Thus $mathrm{rank}(A)= n-1$.



      3)Yes. If $mathrm{adj}(A)=0$ then all $(n−1)×(n−1)$ minors of $A$ are zero, hence $mathrm{rank}(A)leq 2$






      share|cite|improve this answer











      $endgroup$
















        1












        1








        1





        $begingroup$

        1)Yes. If $mathrm{rank}(mathrm{adj}(A))=n$ then $mathrm{adj}(A)$ is invertible, so $A=mathrm{det}(A)mathrm{adj}(A)^{-1}$ is invertible, because it can not be zero.



        2)Yes. If $mathrm{rank}(mathrm{adj}(A))=1$ then $A$ is not invertible so $mathrm{rank}(A)leq n-1$ but by 3) we can not have $mathrm{rank}(A)< n-1$, since it would imply $mathrm{adj}(A)=0$. Thus $mathrm{rank}(A)= n-1$.



        3)Yes. If $mathrm{adj}(A)=0$ then all $(n−1)×(n−1)$ minors of $A$ are zero, hence $mathrm{rank}(A)leq 2$






        share|cite|improve this answer











        $endgroup$



        1)Yes. If $mathrm{rank}(mathrm{adj}(A))=n$ then $mathrm{adj}(A)$ is invertible, so $A=mathrm{det}(A)mathrm{adj}(A)^{-1}$ is invertible, because it can not be zero.



        2)Yes. If $mathrm{rank}(mathrm{adj}(A))=1$ then $A$ is not invertible so $mathrm{rank}(A)leq n-1$ but by 3) we can not have $mathrm{rank}(A)< n-1$, since it would imply $mathrm{adj}(A)=0$. Thus $mathrm{rank}(A)= n-1$.



        3)Yes. If $mathrm{adj}(A)=0$ then all $(n−1)×(n−1)$ minors of $A$ are zero, hence $mathrm{rank}(A)leq 2$







        share|cite|improve this answer














        share|cite|improve this answer



        share|cite|improve this answer








        edited Dec 30 '18 at 16:10

























        answered Dec 30 '18 at 15:44









        moutheticsmouthetics

        52137




        52137























            0












            $begingroup$

            If by adjoint of $A$ you mean adjoint operator, i.e. $operatorname{adj}(A) = A^*$ then your claimed properties are not true. For instance, any Hermetian matrix satisfies $A=A^*$ so the rank of $A$ is the rank of $A^*$.



            More generally, for any matrix $A$, the conjugate transpose $A^*$ always has the same rank.



            As an explicit example, consider the $3times 3$ matrix:
            $$
            A = begin{bmatrix}
            1 & 0 & 0 \
            0 & 1 & 0 \
            0 & 0 & 0
            end{bmatrix}
            $$



            This is clearly rank $2$, but the conjugate transpose is also rank 2.



            That said, if there exists some definition of "adjoint" satisfying the 3 properties you stated, then it is true that $operatorname{rank}operatorname{adj}(A)=1$ implies $operatorname{rank}(A) = n-1$. To see this, note that if $operatorname{rank}(A)$ did not equal $n-1$ then $operatorname{rank}operatorname{adj}(A)neq1$






            share|cite|improve this answer











            $endgroup$













            • $begingroup$
              $mathrm{adj}(A)$ is the transpose of the cofactor matrix of $A$ which make it different from $A^*$ the transpose of the conjugate.
              $endgroup$
              – mouthetics
              Dec 31 '18 at 15:15
















            0












            $begingroup$

            If by adjoint of $A$ you mean adjoint operator, i.e. $operatorname{adj}(A) = A^*$ then your claimed properties are not true. For instance, any Hermetian matrix satisfies $A=A^*$ so the rank of $A$ is the rank of $A^*$.



            More generally, for any matrix $A$, the conjugate transpose $A^*$ always has the same rank.



            As an explicit example, consider the $3times 3$ matrix:
            $$
            A = begin{bmatrix}
            1 & 0 & 0 \
            0 & 1 & 0 \
            0 & 0 & 0
            end{bmatrix}
            $$



            This is clearly rank $2$, but the conjugate transpose is also rank 2.



            That said, if there exists some definition of "adjoint" satisfying the 3 properties you stated, then it is true that $operatorname{rank}operatorname{adj}(A)=1$ implies $operatorname{rank}(A) = n-1$. To see this, note that if $operatorname{rank}(A)$ did not equal $n-1$ then $operatorname{rank}operatorname{adj}(A)neq1$






            share|cite|improve this answer











            $endgroup$













            • $begingroup$
              $mathrm{adj}(A)$ is the transpose of the cofactor matrix of $A$ which make it different from $A^*$ the transpose of the conjugate.
              $endgroup$
              – mouthetics
              Dec 31 '18 at 15:15














            0












            0








            0





            $begingroup$

            If by adjoint of $A$ you mean adjoint operator, i.e. $operatorname{adj}(A) = A^*$ then your claimed properties are not true. For instance, any Hermetian matrix satisfies $A=A^*$ so the rank of $A$ is the rank of $A^*$.



            More generally, for any matrix $A$, the conjugate transpose $A^*$ always has the same rank.



            As an explicit example, consider the $3times 3$ matrix:
            $$
            A = begin{bmatrix}
            1 & 0 & 0 \
            0 & 1 & 0 \
            0 & 0 & 0
            end{bmatrix}
            $$



            This is clearly rank $2$, but the conjugate transpose is also rank 2.



            That said, if there exists some definition of "adjoint" satisfying the 3 properties you stated, then it is true that $operatorname{rank}operatorname{adj}(A)=1$ implies $operatorname{rank}(A) = n-1$. To see this, note that if $operatorname{rank}(A)$ did not equal $n-1$ then $operatorname{rank}operatorname{adj}(A)neq1$






            share|cite|improve this answer











            $endgroup$



            If by adjoint of $A$ you mean adjoint operator, i.e. $operatorname{adj}(A) = A^*$ then your claimed properties are not true. For instance, any Hermetian matrix satisfies $A=A^*$ so the rank of $A$ is the rank of $A^*$.



            More generally, for any matrix $A$, the conjugate transpose $A^*$ always has the same rank.



            As an explicit example, consider the $3times 3$ matrix:
            $$
            A = begin{bmatrix}
            1 & 0 & 0 \
            0 & 1 & 0 \
            0 & 0 & 0
            end{bmatrix}
            $$



            This is clearly rank $2$, but the conjugate transpose is also rank 2.



            That said, if there exists some definition of "adjoint" satisfying the 3 properties you stated, then it is true that $operatorname{rank}operatorname{adj}(A)=1$ implies $operatorname{rank}(A) = n-1$. To see this, note that if $operatorname{rank}(A)$ did not equal $n-1$ then $operatorname{rank}operatorname{adj}(A)neq1$







            share|cite|improve this answer














            share|cite|improve this answer



            share|cite|improve this answer








            edited Dec 30 '18 at 16:41

























            answered Dec 30 '18 at 16:29









            tchtch

            833310




            833310












            • $begingroup$
              $mathrm{adj}(A)$ is the transpose of the cofactor matrix of $A$ which make it different from $A^*$ the transpose of the conjugate.
              $endgroup$
              – mouthetics
              Dec 31 '18 at 15:15


















            • $begingroup$
              $mathrm{adj}(A)$ is the transpose of the cofactor matrix of $A$ which make it different from $A^*$ the transpose of the conjugate.
              $endgroup$
              – mouthetics
              Dec 31 '18 at 15:15
















            $begingroup$
            $mathrm{adj}(A)$ is the transpose of the cofactor matrix of $A$ which make it different from $A^*$ the transpose of the conjugate.
            $endgroup$
            – mouthetics
            Dec 31 '18 at 15:15




            $begingroup$
            $mathrm{adj}(A)$ is the transpose of the cofactor matrix of $A$ which make it different from $A^*$ the transpose of the conjugate.
            $endgroup$
            – mouthetics
            Dec 31 '18 at 15:15


















            draft saved

            draft discarded




















































            Thanks for contributing an answer to Mathematics Stack Exchange!


            • Please be sure to answer the question. Provide details and share your research!

            But avoid



            • Asking for help, clarification, or responding to other answers.

            • Making statements based on opinion; back them up with references or personal experience.


            Use MathJax to format equations. MathJax reference.


            To learn more, see our tips on writing great answers.




            draft saved


            draft discarded














            StackExchange.ready(
            function () {
            StackExchange.openid.initPostLogin('.new-post-login', 'https%3a%2f%2fmath.stackexchange.com%2fquestions%2f3056904%2fconverse-of-statements-about-the-rank-of-adjugate-matrices%23new-answer', 'question_page');
            }
            );

            Post as a guest















            Required, but never shown





















































            Required, but never shown














            Required, but never shown












            Required, but never shown







            Required, but never shown

































            Required, but never shown














            Required, but never shown












            Required, but never shown







            Required, but never shown







            Popular posts from this blog

            Biblatex bibliography style without URLs when DOI exists (in Overleaf with Zotero bibliography)

            ComboBox Display Member on multiple fields

            Is it possible to collect Nectar points via Trainline?